Saltar al contenido principal
LibreTexts Español

5.6: Problemas

  • Page ID
    128972
  • \( \newcommand{\vecs}[1]{\overset { \scriptstyle \rightharpoonup} {\mathbf{#1}} } \)

    \( \newcommand{\vecd}[1]{\overset{-\!-\!\rightharpoonup}{\vphantom{a}\smash {#1}}} \)

    \( \newcommand{\id}{\mathrm{id}}\) \( \newcommand{\Span}{\mathrm{span}}\)

    ( \newcommand{\kernel}{\mathrm{null}\,}\) \( \newcommand{\range}{\mathrm{range}\,}\)

    \( \newcommand{\RealPart}{\mathrm{Re}}\) \( \newcommand{\ImaginaryPart}{\mathrm{Im}}\)

    \( \newcommand{\Argument}{\mathrm{Arg}}\) \( \newcommand{\norm}[1]{\| #1 \|}\)

    \( \newcommand{\inner}[2]{\langle #1, #2 \rangle}\)

    \( \newcommand{\Span}{\mathrm{span}}\)

    \( \newcommand{\id}{\mathrm{id}}\)

    \( \newcommand{\Span}{\mathrm{span}}\)

    \( \newcommand{\kernel}{\mathrm{null}\,}\)

    \( \newcommand{\range}{\mathrm{range}\,}\)

    \( \newcommand{\RealPart}{\mathrm{Re}}\)

    \( \newcommand{\ImaginaryPart}{\mathrm{Im}}\)

    \( \newcommand{\Argument}{\mathrm{Arg}}\)

    \( \newcommand{\norm}[1]{\| #1 \|}\)

    \( \newcommand{\inner}[2]{\langle #1, #2 \rangle}\)

    \( \newcommand{\Span}{\mathrm{span}}\) \( \newcommand{\AA}{\unicode[.8,0]{x212B}}\)

    \( \newcommand{\vectorA}[1]{\vec{#1}}      % arrow\)

    \( \newcommand{\vectorAt}[1]{\vec{\text{#1}}}      % arrow\)

    \( \newcommand{\vectorB}[1]{\overset { \scriptstyle \rightharpoonup} {\mathbf{#1}} } \)

    \( \newcommand{\vectorC}[1]{\textbf{#1}} \)

    \( \newcommand{\vectorD}[1]{\overrightarrow{#1}} \)

    \( \newcommand{\vectorDt}[1]{\overrightarrow{\text{#1}}} \)

    \( \newcommand{\vectE}[1]{\overset{-\!-\!\rightharpoonup}{\vphantom{a}\smash{\mathbf {#1}}}} \)

    \( \newcommand{\vecs}[1]{\overset { \scriptstyle \rightharpoonup} {\mathbf{#1}} } \)

    \( \newcommand{\vecd}[1]{\overset{-\!-\!\rightharpoonup}{\vphantom{a}\smash {#1}}} \)

    5.5 Desacoplamiento quantal hamiltonianos

    Demostrar el teorema de “desacoplamiento Hamilton implica factorizar la función de partición” de la sección 5.1.2 para sistemas cuánticos.

    5.6 Anomalía de Schottky

    Una molécula puede ser modelada con precisión por un sistema quantal de dos estados con energía de estado fundamental 0 y energía de estado excitado\( \epsilon\). Demostrar que el calor específico interno es

    \[ c_{V}^{\mathrm{int}}(T)=k_{B}\left(\frac{\epsilon}{k_{B} T}\right)^{2} \frac{e^{-\epsilon / k_{B} T}}{\left(1+e^{-\epsilon / k_{L} T}\right)^{2}}\]

    Esbozar este calor específico en función de\( k_B / \epsilon\). ¿Cómo se comporta la función cuándo\( k_B T \ll \epsilon\) y\( k_B T \gg \epsilon \)?

    5.7 Oscilador armónico simple

    Supongamos que una molécula puede modelarse con precisión como un oscilador armónico de frecuencia natural ω.

    a. encontrar la energía interna esperada de una de esas moléculas, escrita como una suma de la energía del estado fundamental más una parte dependiente de la temperatura.

    b. Demostrar que el calor específico interno es

    \[ c_{V}^{\mathrm{int}}(T)=k_{B}\left(\frac{\hbar \omega}{k_{B} T}\right)^{2} \frac{e^{-\hbar \omega / k_{B} T}}{\left(1-e^{-\hbar \omega / k_{H} T}\right)^{2}}.\]

    c. Demostrar que a bajas temperaturas\( \left(k_{B} T \ll \hbar \omega\right)\),

    \[ c_{V}^{\mathrm{int}}(T) \approx k_{B}\left(\frac{\hbar \omega}{k_{B} T}\right)^{2} e^{-\hbar \omega / k_{B} T}\]

    mientras que a altas temperaturas\( \left(k_{B} T \rr \hbar \omega\right)\),

    \[ c_{V}^{\mathrm{int}}(T) \approx k_{B}.\]

    d. (Opcional.) Demostrar que la corrección quantal principal al calor específico de alta temperatura es

    \[ c_{V}^{\mathrm{int}}(T)=k_{B}\left[1-\frac{1}{12} x^{2}+\mathcal{O}\left(x^{3}\right)\right], \quad \text { where } \quad x=\frac{\hbar \omega}{k_{B} T}.\]

    e. Esbozar el calor específico interno en función de\(k_B T / \hbar \omega \).

    5.8 Oscilador armónico simple — entropía

    Estaba hablando con alguien que afirmaba que la termodinámica no se aplicaba a los sistemas de átomos que interactúan, ni a los seres vivos, ni a nada que no sea el gas ideal. Me desafió “¿Cuál es la entropía de un péndulo? ¡La respuesta es que la entropía no se aplica a un péndulo!” Responde su pregunta para un péndulo en la aproximación simple del oscilador armónico, con frecuencia natural ω y temperatura T.

    5.9 Energía cinética en el aire

    ¿Cuál es la energía cinética es un litro cúbico de aire (principalmente nitrógeno y oxígeno) a una atmósfera de presión y temperatura ambiente? A una atmósfera presión y temperatura 600 K? Utiliza la mecánica clásica, y expresa tu respuesta tanto en Julios como en la unidad comercial kilovatios-hora. (Clue: Si calculas la energía cinética promedio de una molécula, estás trabajando demasiado duro). ¿Por qué nadie explota toda esta valiosa energía que hay por ahí en cada metro cúbico de aire?

    5.10 Comparación conceptual

    a. Explicar cualitativamente por qué los resultados de los dos problemas anteriores son paralelos a bajas temperaturas.

    b. (Más difícil.) Explicar cualitativamente ambos resultados de alta temperatura. (Clue: A altas temperaturas, la energía promedio por partícula en el caso Schottky se aproxima\(\epsilon\) /2. ¿Por qué?)

    5.11 Compresibilidad de un gas diatómico

    Encuentra la compresibilidad isotérmica κ T para un gas diatómico ideal, donde cada molécula se modela como una mancuerna con momento de inercia I.

    5.12 Sistemas con un pequeño número de estados

    (¡Este problema no requiere cálculo! Todas las respuestas se pueden encontrar en tu cabeza.) Una colección de partículas que no interactúan está en equilibrio térmico. Cada partícula tiene solo tres valores propios de energía, a saber 0,\(\epsilon\), y\(4 \epsilon\).

    a. ¿Cuál es el criterio de “alta temperatura” en esta situación?

    b. Supongamos que hay tres autoestados energéticos no degenerados. A altas temperaturas, ¿cuál es la energía promedio de cada partícula? (pista: La respuesta no es\(4\epsilon\).)

    c. Ahora supongamos que los dos estados propios energéticos inferiores son no degenerados, pero que hay dos estados independientes con energía\(4 \epsilon\). ¿Cuál es la energía promedio por partícula a altas temperaturas en este caso?

    5.13 Oscilador anarmónico

    Los valores propios de energía de un oscilador armónico simple están igualmente espaciados, y hemos explorado las consecuencias de esto para la capacidad calorífica de una colección de osciladores armónicos. Supongamos que un oscilador anarmónico es aproximadamente armónico (con frecuencia natural ω 0) para energías pequeñas, pero que para energías grandes (mayores que, digamos, E x) los valores propios se espacian más estrechamente a medida que aumenta la energía. A temperaturas superiores a E x/k B, ¿la capacidad calorífica de una colección de tales osciladores anarmónicos será mayor o menor que la de una colección de osciladores armónicos con la misma frecuencia natural ω 0? ¿Por qué?

    5.14 Características descriptivas de los modelos

    (Este problema es robado de una prueba GRE Physics.)

    Dos modelos posibles para un gas ideal diatómico son la mancuerna rígida (modelo R; partículas de dos puntos conectadas por una varilla rígida) y la mancuerna elastica (modelo S; partículas de dos puntos conectadas por un resorte). En la mecánica estadística clásica, ¿cuál de las siguientes afirmaciones es cierta?

    a. El modelo R tiene un calor específico\(c_V = \frac{3}{2} k_B\).

    b. El modelo S tiene un calor específico menor que el modelo R.

    c. El Modelo S siempre es correcto.

    d. El modelo R siempre es correcto.

    e. La elección entre los modelos R y S depende de la temperatura.

    5.15 Un sistema n-estado, cualitativamente

    Una molécula modelo tiene n niveles de energía equiespaciados, todos ellos no degenerados, con espaciamiento de energía\( \epsilon\). Así como n varía de 2 a ∞ este modelo interpola entre el sistema Schottky del problema 5.6 y el oscilador armónico simple del problema 5.7.

    a. Encuentre una aproximación a baja temperatura para el calor específico que sea independiente de n.

    b. A altas temperaturas, el calor específico se aproxima a cero. ¿Cuál es el criterio de “alta temperatura”?

    c. A altas temperaturas, ¿cuál es la energía esperada de este modelo?

    d. Existe un teorema que afirma que a cualquier temperatura positiva fija, el calor específico debe aumentar al aumentar n. Supongamos este teorema y úsalo para demostrar que a medida que n aumenta, el máximo en la curva específica de calor versus temperatura se vuelve mayor.

    5.16 Un sistema de estado n, cuantitativamente

    Demostrar que el sistema del problema anterior tiene calor específico interno

    \[ c_{V}^{\mathrm{int}}(T)=k_{B}\left[\left(\frac{\epsilon}{k_{B} T}\right)^{2} \frac{e^{-\epsilon / k_{B} T}}{\left(1-e^{-\epsilon / k_{B} T}\right)^{2}}-\left(\frac{n \epsilon}{k_{B} T}\right)^{2} \frac{e^{-n \epsilon / k_{B} T}}{\left(1-e^{-n \epsilon / k_{B} T}\right)^{2}}\right].\]

    ¿Esta expresión tiene los límites adecuados cuando n = 2 y cuando n → ∞?


    This page titled 5.6: Problemas is shared under a CC BY-SA license and was authored, remixed, and/or curated by Daniel F. Styer.